Sunteți pe pagina 1din 62

SAINT LOUIS UNIVERSITY SCHOOL OF LAW

Case Digests Compendium


Natural Resources Law and Environmental Law
LLB 2-A

Professor: Atty. Jerico G. Gay-ya


1

Table of Contents
Pedro Lee Hong Hok et al. vs. Aniano David et al. .............................................................. 2
Collado v. CA .......................................................................................................................... 4
NESTOR PAGKATIPUNAN, et. al. vs COURT OF APPEALS ...................................... 6
Cariño v. CA ............................................................................................................................ 9
BARLIN vs. RAMIREZ ....................................................................................................... 11
Cruz v. SENR ........................................................................................................................ 13
FRANCISCO I. CHAVEZ vs PUBLIC ESTATES AUTHORITY and AMARI
COASTAL BAY DEVELOPMENT CORPORATION .................................................... 16
FRANCISCO I. CHAVEZ vs PUBLIC ESTATES AUTHORITY and AMARI
COASTAL BAY DEVELOPMENT CORPORATION .................................................... 18
SENR, et. al. v. Yap, et. al., .................................................................................................. 21
Nancy T. Lorzano v. Juan Tabayag, Jr. ............................................................................. 25
Heirs of Reterta v. Spouses Mores and Lopez .................................................................... 27
REPUBLIC OF THE PHILIPPINES vs. HEIRS OF AGUSTIN L. ANGELES ............ 29
CARO v. SUCALDITO ........................................................................................................ 31
DE DELGADO VS. COURT OF APPEALS ..................................................................... 33
HEIRS OF VALERIANO S. CONCHA v. SPOUSES LUMOCSO................................. 36
Cawis, et al. v. Cerilles, et al................................................................................................. 38
Saad, etc. v. Republic of the Philippines ............................................................................. 40
Heirs of Nagaño v Court of Appeals ................................................................................... 43
Dolar v. Barangay Lublub etc.............................................................................................. 44
East Asia Traders, Inc. v Republic of the Philippines, represented by the Director,
Lands Management Bureau ................................................................................................. 46
Estate of the Late Yujuico v. Republic of the Philippine .................................................. 48
Sta. Ignacia Rural Bank, Inc. v. Court of Appeals ............................................................ 50
Mejia v. Gabayan, et. al. ....................................................................................................... 52
Kings Properties Corp. v. Galido ........................................................................................ 55
Republic of the Philippines vs. Court of Appeals............................................................... 57
LOPEZ V. CA ....................................................................................................................... 59
2

Pedro Lee Hong Hok et al. vs. Aniano David et al.


G.R No. L- 30389
December 27, 1972

Doctrine: The acquisition of free patent provided under Commonwealth Act No. 141, as
certified by Director of Lands and approved by the Secretary of Agriculture and Natural
Resources

Facts:
On October 21, 1959 private respondent David acquired lawful title to the disputed land
pursuant to his miscellaneous sales application in accordance with which an order of award
and for issuance of a sales patent was made by the Director of Lands on June 18, 1958,
covering Lot 2892 containing an area of 226 square meters, which is a portion of Lot 2863 of
the Naga Cadastre. On the basis of the order of award of the Director of Lands the
Undersecretary of Agriculture and Natural Resources issued on August 26, 1959,
Miscellaneous Sales Patent no. v-1209 pursuant to which OCT No. 510 which was issued by
the Register of Deeds of Naga City. The petitioners filed an action to have the private
respondent's Torrens Title be declared null and void. They base their claim of ownership of
the disputed lot through accretion which was rejected by the Court of Appeals since during
the filing of the sales application of respondent David and during all the proceedings in
connection with said application, up to the actual issuance of the sales patent in his favor, the
petitioners did not put up any opposition or adverse claim thereto.

Issue: Can the petitioner have the title of the respondent be declared null and void.

Held: No. Only the Government, represented by the Director of Lands, or the Secretary of
Agriculture and Natural Resources, can bring an action to cancel a void certificate of title
issued pursuant to a void patent. This was not done by said officers but by private parties like
the petitoners, who cannot claim that the patent and title issued for the land involved are void
since they are not the registered owners thereof nor had they been declared as owners in the
cadastral proceedings of Naga Cadastre after claiming it as their private property. The fact
that the grant was made by the government is undisputed. Whether the grant was in
conformity with the law or not is a question which the government may raise, but until it is
raised by the government and set aside, the defendant cannot question it. The legality of the
grant is a question between the grantee and the government. The distinction in public law
between the government authority possessed by the state which is appropriately embraced in
the concept of sovereignty, and its capacity to own or acquire property, the former comes
under the heading of imperium and the latter of dominium. The use of this term is
appropriate with reference to lands held by the state in its proprietary character. In such
3

capacity, it may provide for the exploitation and use of lands and other natural resources,
including their disposition, except as limited by the Constitution.

There being no evidence whatever that the property in question was ever acquired by the
applicants or their ancestors either by composition title from the Spanish Government or by
possessory information title or by any other means for the acquisition of public lands, the
property must be held to be public domain. For it is well-settled that no public land can be
acquired by private persons without any grant, express or implied, from the government. It is
indispensable then that there be a showing of a title from the state or any other mode of
acquisition recognized by law.
4

Collado v. CA
G. R. No. 107764
October 4, 2002

“Regalian Doctrine”

Facts:

On April 25, 1985, petitioner Edna T. Collado filed with the land registration court an
application for registration of a parcel of land with an approximate area of 1,200,766 square
meters or 120.0766 hectares. The Lot is situated inside Mariquina Watershed Reservation.

Petitioners alleged that they have occupied the Lot since time immemorial. Their
possession has been open, public, notorious and in the concept of owners. Having presented
sufficient evidence, the trial court approved of their application for land registration.

On August 6, 1991 the Solicitor General filed with the Court of Appeals to annul the
judgment on the ground that there had been no clear showing that the Lot had been
previously classified as alienable and disposable making it subject to private appropriation.
The Court of Appeals declared petitioner’s registration null and void, stating that Collado
failed to present evidence that the land applied for has been segregated from the public
domain and declared alienable and disposable.

Collado points out that EO 33 contains a saving clause that the reservations are subject
to existing private rights, if any there be. Petitioners contend that their claim of ownership
goes all the way back to 1902, when their known predecessor-in-interest, Sesinando Leyva,
laid claim and ownership over the Lot. They claim that the presumption of law then
prevailing under the Philippine Bill of 1902 and Public Land Act No. 926 was that the land
possessed and claimed by individuals as their own are agricultural lands and therefore
alienable and disposable. They conclude that private rights were vested on Sesinando Leyva
before the issuance of EO 33, thus excluding the Lot from the Marikina Watershed
Reservation.

Issue:

Can petitioner Collado validly claim ownership of the said parcel of land?

Held:

No. Petitioner Collado cannot validly claim ownership of the said parcel of land.
5

1. Under the Regalian Doctrine, all lands not within private ownership are presumed to
belong to the State.
Act No. 926, the Public Land Act, operated on the assumption that title to public
lands in the Philippine Islands remained in the government; and that the governments
title to public land sprung from the Treaty of Paris and other subsequent treaties
between Spain and the United States. Thus, it is plain error for petitioners to argue
that under the Philippine Bill of 1902 and Public Land Act No. 926, mere possession
by private individuals of lands creates the legal presumption that the lands are
alienable and disposable.

2. Assuming that the Lot was alienable and disposable land prior to the issuance of EO
33 in 1904, EO 33 reserved the Lot as a watershed. Since then, the Lot became non-
disposable and inalienable public land. At the time petitioners filed their application
on April 25, 1985, the Lot has been reserved as a watershed under EO 33 for 81
years prior to the filing of petitioners application.

The period of occupancy after the issuance of EO 33 in 1904 could no longer be


counted because as a watershed reservation, the Lot was no longer susceptible of
occupancy, disposition, conveyance or alienation. Section 48 (b) of CA 141, as
amended, applies exclusively to alienable and disposable public agricultural
land. Forest lands, including watershed reservations, are excluded.

3. The possession of public land, however long the period may have extended, never
confers title thereto upon the possessor because the statute of limitations with regard
to public land does not operate against the State, unless the occupant can prove
possession and occupation of the same under claim of ownership for the required
number of years to constitute a grant from the State.

4. A positive act (e.g., an official proclamation) of the Executive Department is needed


to declassify land which had been earlier classified as a watershed reservation and to
convert it into alienable or disposable land for agricultural or other purposes. The lot
was never reclassified as alienable and disposable land. Hence, the lot, being a
watershed reservation, cannot be disposed of or alienated.
6

NESTOR PAGKATIPUNAN, et. al. vs COURT OF APPEALS


G.R. No. 129682.
March 21, 2002

DOCTRINE:
Under the Regalian doctrine, all lands of the public domain belong to the
State, and the State is the source of any asserted right to ownership in land and charged with
the conservation of such patrimony. This same doctrine also states that all lands not
otherwise appearing to be clearly within private ownership are presumed to belong to
the State. To overcome such presumption, incontrovertible evidence must be shown by the
applicant that the land subject of the application is alienable or disposable.

RELEVANT FACTS:
The spouses Getulio Pagkatipunan and Lucrecia Esquires, filed with the CFI of
Gumaca, Quezon an application for judicial confirmation and registration of their title
to Lots 1 and 2 of Plan Psu-174406 and Lots 1 and 2 of Plan Psu-112066, all
located in San Narciso, Quezon. On May 4, 1961, the CFI entered an order of default
against the whole world, except spouses Felicisimo Almace and Teodulo Medenilla
who were given 10 days to file their written opposition as regards Lot No. 2 of Plan Psu-
174406. Upon motion of petitioners predecessors, Lot No. 2 of Plan Psu-174406 was
removed from the coverage of the application. On June 15, 1967, the Court of First
Instance promulgated a decision confirming petitioners title to the property. On October
23, 1967, OCT No. O-12665 was issued in the name of petitioners.
Almost 18 years later, or on September 12, 1985, the Republic of the Philippines filed
with the IAC an action to declare the proceedings in LRC Case No. 91-G, LRC Record No.
N-19930 before the CFI of Gumaca, Quezon null and void, and to cancel OCT No. 0-
12665 and titles derived therefrom as null and void, to direct the register of deeds to annul
said certificates of title, and to confirm the subject land as part of the public domain. The
Republic claimed that at the time of filing of the land registration case and of rendition of the
decision on June 15, 1967, the subject land was classified as timberland under LC Project
No. 15-B of San Narciso, Quezon, as shown in BF Map No. LC-1180, hence inalienable
and not subject to registration. Moreover, petitioners title thereto cannot be confirmed for
lack of showing of possession and occupation of the land in the manner and for the
length of time required by Section 48(b), CA No. 141, as amended. Neither did petitioners
have any fee simple title which may be registered under Act No. 496, as amended.
Consequently, the CFI did not acquire jurisdiction over the res and any proceedings had
therein were null and void.
7

Petitioners raised the special defenses of indefeasibility of title and res judicata. They
argued that due to the lapse of a considerable length of time, the judgment of the CFI of
Quezon in the land registration case has become final and conclusive against the Republic.
Moreover, the action for reversion of the land to the public domain is barred by prior
judgment.
The IAC held that the land in question was forestral land, hence not registrable.
There was no evidence on record to show that the land was actually and officially delimited
and classified as alienable or disposable land of the public domain. Therefore, the CFI did
not acquire jurisdiction to take cognizance of the application for registration and to
decide the same. Consequently, the action to declare null and void the June 15, 1967
decision for lack of jurisdiction did not prescribe.
Petitioners moved for the reconsideration of the afore-cited decision reiterating that
the land in question was agricultural because it was possessed and cultivated as such
long before its classification as timberland by the Bureau of Forestry in 1955. Petitioners
and their predecessors-in-interest have been in open, continuous, exclusive, notorious
possession and occupation of said land for agricultural and cattle raising purposes as far
back as the Spanish regime. Following the doctrine in Oracoy v. Director of Lands, private
interest had intervened and petitioners acquired vested rights which can no longer be
impaired by the subsequent classification of the land as timberland by the Director of
Forestry.

ISSUE/S:
1. Is the petitioners correct in claiming that their title to the land became
incontrovertible and indefeasible 1 year after issuance of the decree of registration,
barring the Republics cause of action due to prescription and res judicata?

2. Is the petitioners correct in claiming that the land is agricultural and the vested
rights acquired by petitioners predecessors-in-interest who have been in open,
continuous, adverse and public possession of the land in question since time
immemorial and for more than 30 years prior to the filing of the application for
registration in 1960 cannot be impaired by the classification of the Bureau of
Forestry in 1955 as timberland?

HOLDING/S:
1. NO. Petitioners contention that the Republic is now barred from questioning the
validity of the certificate of title issued to them considering that it took the
government almost 18 years to assail the same is erroneous. It is a basic precept that
8

prescription does not run against the State.1 The lengthy occupation of the disputed
land by petitioners cannot be counted in their favor, as it remained part of the
patrimonial property of the State, which is inalienable and indisposable.

2. NO. Unless public land is shown to have been reclassified or alienated to a private
person by the State, it remains part of the inalienable public domain. Occupation
thereof in the concept of owner, no matter how long, cannot ripen into ownership and
be registered as a title.2 Before any land may be declassified from the forest group
and converted into alienable or disposable land for agricultural or other purposes,
there must be a positive act from the government. Even rules on the confirmation of
imperfect titles do not apply unless and until the land classified as forest land is
released in an official proclamation to that effect so that it may form part of the
disposable agricultural lands of the public domain. Declassification of forest
land is an express and positive act of Government. It cannot be presumed.
Neither should it be ignored nor deemed waived.
Thus, it is clear that the applicant must prove not only his open, continuous,
exclusive and notorious possession and occupation of the land either since time
immemorial or for the period prescribed therein, but most importantly, he must
prove that the land is alienable public land. In the case at bar, petitioners failed to do
so since there was no evidence showing that the land has been reclassified as
disposable or alienable.
Evidence extant on record would also show that at the time of filing of the
application for land registration and issuance of the certificate of title over the
disputed land in the name of petitioners, it was classified as timberland and formed
part of the public domain, according to the certification issued by the Bureau of
Forest Development on April 1, 1985. Petitioners even admitted this fact during the
proceedings on March 10, 1986, when they confirmed that the land has been
classified as forming part of forest land, although only on August 25, 1955. Since no
imperfect title can be confirmed over lands not yet classified as disposable or
alienable, the title issued to herein petitioners is considered void ab initio.

1Republic v. Court of Appeals, 171 SCRA 721,734 [1989]


2Menguito v. Republic, 348 SCRA 128, 139 [2000], citing De Ocampo v. Arlos, 343 SCRA 716,
724 [2000]
9

Cariño v. CA
212 U.S. 449 (1909)
February 23, 1909

Doctrine:

An exception to the jura regalia doctrine would be any land that should have been in
the possession of an occupant and of his predecessors in interest since time immemorial, for
such possession would justify the presumption that the land had never been part of the public
domain or that it had been a private property even before the Spanish conquest.

Facts:

Cariño and his ancestors are Igorots of the Province of Benguet where the land is
situated. Cariño filed an application to Philippine Court of Land Registration for the
registration of certain land. The application was granted by the court on March 4, 1904. The
State, on behalf of the United States, opposed having taken possession of the property for
public and military purposes. Cariño and his ancestors had held the land as owners. His
grandfather had lived upon it, and had maintained fences sufficient for the holding of cattle,
according to the custom of the country. They all had been recognized as owners by the
Igorots, and he had inherited or received the land from his father in accordance with Igorot
custom. Cariño only presented possessory information and no other documentation.

Issues:

Is Cariño the legal owner of the ancestral land having been the recognized owner by
the Igorots since time immemorial?

Ruling:
Yes, the applicant should be granted what he seeks, and should not be deprived of
what, by the practice and belief of those among whom he lived, was his property. The royal
decree of February 13, 1894 should not be construed as a confiscation, but as the withdrawal
of a privilege. As a matter of fact, the applicant never was disturbed. This same decree is
quoted by the Court of Land Registration for another recognition of the common law
prescription of thirty years as still running against alienable Crown land. The effect of the
registration requirement was not to confer title, but simply to establish it, as already
conferred by the decree, if not by earlier law. The court laid down the presumption of a
certain title held as far back as memory went and under a claim of private ownership. Land
held by this title is presumed to never have been public land. As far back as testimony or
memory goes, the land has been held by individuals under a claim of private ownership, it
10

will be presumed to have been held in the same way from before the Spanish conquest, and
never to have been public land.
11

BARLIN vs. RAMIREZ


G.R. No. L-2832
November 24, 1906

DOCTRINE:
Churches and its adjuncts are property of the Catholic Church and not the State. Since
the property did not belong to the Spanish crown, neither the United States had any right over
it. The properties were beyond the commerce of man. They were not considered public
property, nor could they be subject of private property appropriation in any manner or in the
sense that any private person could be the owner thereof. These properties constituted a
distinctive characteristic of being devoted to the worship of God.

RELEVANT FACTS:
Since 1839, there had been priests of the Roman Catholic Church in the pueblo of
Lagonoy, in the Province of Ambos Camarines. When the church and convent were burned,
an order of the provincial governor mandated the people of the pueblo to work on its
reconstruction. After the completion of the church it was always administered, until 1902, by
a priest of a Roman Catholic Communion and all the people of the pueblo professed that faith
and belonged to that church.
Father Vicente Ramirez, having been appointed by Monsignor Jorge Barlin, took
possession of the church in 1901. Ramirez later decided to affiliate with the new church.
Upon learning that his priest had turned schismatic, Barlin, in his capacity as apostolic
administrator of the vacant see of the archdiocese, sent a new parish priest to the said town
and advised Ramirez to vacate and turn over the property to the proper canonical authority.
Ramirez refused to vacate, in fact seized the property and held that the rightful owner was the
town of Lagonoy.
Barlin brought this issue to the Court of First Instance. The Municipality of Lagonoy
filed a petition asking that it be allowed to intervene in the case and join with the defendant,
Ramirez, as a defendant therein. The petition was granted and the municipality alleged in its
answer that Ramirez was in possession of the property described in the complaint under the
authority and with the consent of the Municipality of Lagonoy and that such municipality
was the owner thereof. The case was tried and decided in favor of the Catholic Church.
Ramirez then brought the case to the Supreme Court by a bill of exceptions.

ISSUE:
Is the subject property wherein the said church and its adjuncts situated not owned by
the government, but by the Roman Catholic Church having juridical personality?
12

HOLDING:

Yes. The Court held that the subject property is owned by the Roman Catholic
Church. There was a law that states that all church buildings were made by the Spanish
government and representatives in the Philippines using government and private local funds,
but the Spanish government implemented this to the effect that the churches and its income
were dedicated for the propagation of the faith. Hence, its properties were beyond the
commerce of man. Priests held them in the concept of guardians or stewards. From the
earliest times, down to the cession of the Philippines to the United States, churches and other
consecrated objects were considered outside of the commerce of man. They were not public
property, nor could they be subjects of private property in the sense that any private person
could the owner thereof. They constituted a kind of property distinctive characteristic of
which was that it was devoted to the worship of God.

Furthermore, the municipality cannot show evidence of title, right of ownership or


possession. Its claim of ownership is rested upon the propositions that the property in
question belonged prior to the Treaty of Paris to the Spanish Government; that by the treaty
of Paris the ownership thereof passed to the Government of the United States then to the
Government of the Philippine Islands, and that by the circular of that Government, the
ownership and the right to the possession of this property passed to the municipality of
Lagonoy. These propositions are false. As a matter of law, the Spanish Government at the
time the treaty of peace was signed, was not the owner of this property, nor of any other
property like it, situated in the Philippine Islands. The Spanish Government in fact expressly
exempted ownership of churches and similar properties in several of its laws. Additionally, if
we should admit that the propositions are true, there was no evidence to support the last
proposition, namely that the Government of the Philippine Islands had transferred the
ownership of this church to the Municipality of Lagonoy.

It was suggested by Barlin that the Roman Catholic Church has no legal personality
in the Philippine Islands. This suggestion, made with reference to an institution which
antedates by almost a thousand years any other personality in Europe, and which existed
"when Grecian eloquence still flourished in Antioch, and when idols were still worshipped in
the temple of Mecca," does not require serious consideration. Since the latter half of the third
century, and more particularly since the year 313, when Constantine, by the edict of Milan,
inaugurated an era of protection for the church, the latter gradually entered upon the exercise
of such rights as were required for the acquisition, preservation, and transmission of property
the same as any other juridical entity under the laws of the Empire.
13

Cruz v. SENR
G.R. No. 135385
December 6, 2000

Doctrine/s: 1. Ancestral lands and domains are considered as private land, and never to have
been part of the public domain.
2. Native title to land, or ownership of land by Filipinos by virtue of possession
under a claim of ownership since time immemorial and independent of any grant from the
Spanish Crown is an exception to the theory of jura regalia.

Facts:
In 1997, RA 8371 or the Indigenous Peoples Rights Act (IPRA) was passed.
Isagani Cruz and Cesar Europafiled a petition for prohibition and mandamus, questioning the
constitutionality of certain provisionsof IPRA. They contend that the IPRA allows the
indigenous people/cultural community to own natural resources and that it defines ancestral
lands and ancestral domains in such a way that it may include private landsowned by other
individuals. They also argue that the IPRA categorizes ancestral lands and domains held by
native title as never to have been public land and that allowing the NCIP (National
Commission on Indigenous Peoples) to take jurisdiction over indigenous people land
disputes and making customary law apply to these disputes violates the due process clause.
In the first deliberation before the Supreme Court the results were seven in favor and seven
against (7-7).

Issues:

1.) Is ownership over ancestral lands and ancestral domains by indigenous


peoples/ cultural communities (IP/ICC) provided under the IPRA contravenes
the Regalian Doctrine under the Constitution?
2.) Does the IPRA grant ownership over natural resources found on ancestral lands
and ancestral domains to Ips/ICCs, thereby violating the Constitution?

Held:
As the votes were equally divided (7 to 7) and the necessary majority was not
obtained, the case was redeliberated upon. However, after redeliberation, the voting remained
the same (7-7). Pursuant to Rule 56, Section 7 of the Rules of Civil Procedure, the petition is
dismissed. Justices Puno, Kapunan, and Mendoza wrote separate opinions in dismissing the
petition while Justices Vitug and Panganiban wrote separate opinions in granting the petition.

In affirming the IPRA, Justices Kapunan and Puno answered the issues as follows:
14

1.) No, IPs/ICCs ownership over ancestral lands or ancestral domains does not
violate the Regalian Doctrine. Ancestral lands and domains are considered as
private land, and never to have been part of the public domain. Both Justices
also cited the case of Cario vs. Insular Government which recognized the
existence of native title to the land or ownership of land by Filipinos by virtue
of possession under a claim of ownership since time immemorial.
Both Justices asserted that native title to land, or ownership of land by Filipinos
by virtue of possession under a claim of ownership since time immemorial and
independent of any grant from the Spanish Crown is an exception to the theory
of jura regalia.
According to Justice Puno, the rights of the ICCs/IPs to their ancestral domains
and ancestral lands may be acquired in two modes: (1) by native title over both
ancestral lands and domains; or (2) by torrens title with respect to ancestral
lands only. The IPRA categorically declares ancestral lands and domains held
by native title as never to have been public land and are presumed private.
Native title refers to ICCs/IPs' preconquest rights to lands and domains held
under a claim of private ownership as far back as memory reaches. These lands
are deemed never to have been public lands and are indisputably presumed to
have been held that way since before the Spanish Conquest. These lands
claimed by the IPs have long been theirs by virtue of native title. Moreover, the
option of securing a Torrens Title to the ancestral land indicates that the land is
private.

2. No, the IPRA is not unconstitutional as it does not grant ownership over
natural resources found on ancestral lands and ancestral domains to IPs/ICCs.
Both Justices agreed that natural resources found within ancestral lands and
ancestral domains still belong to the State.
According to Justice Puno, the IPs claims are limited to lands, bodies of water
traditionally and actually occupied by ICCs/IPs, sacred places, traditional
hunting and fishing grounds, and all improvements made by them at any time
within the domains. The IPRA did not mention that the IPs also own all the
other natural resources found within theancestral domains. Moreover, the rights
granted by the IPRA to the ICCs/IPs over the natural resources in their
ancestral domains merely gives the ICCs/IPs, as owners and occupants of the
land on which the resources are found, the right to the small-scale utilization of
these resources, and at the same time, a priority in their large-scale
development and exploitation.
15

Justice Kapunan similarly contends that Section 3(a) of the IPRA which
allegedly divest ownership of natural resources withing ancestral lands and
domains in favour of Ips/ICCs serves only as a yardstick which points out what
properties are within the ancestral domains. It does not confer or recognize any
right of ownership over the natural resources to the indigenous peoples. Its
purpose is definitional and not declarative of a right or title.

Justice Mendoza dismissed the Petition maintaining the purpose of the suit is not to
enforce a property right of petitioners against the government and other
respondents or to demand compensation for injuries suffered by them as a result of
the enforcement of the law, but only to settle what they believe to be the doubtful
character of the law in question. In essence, the petitioners has no standing since
the IPRA does not apply to them except in general and in common with other
citizens.

In granting the petition, Justices Vitug declared the subject provisions of the IPRA
are beyond the context of the fundamental law and virtually amount to an undue
delegation, if not an unacceptable abdication, of State authority over a significant
area of the country and its patrimony and is violative of the Regalian Doctrine. He
also stated that the doctrine laid down in the case of Carino v. Insular Government
cannot override the collective will of the people expressed in the Constitution
which embraces the Regalian Doctrine.

Justice Panganiban voted to grant the petition holds that the Constitution is the
fundamental law of the land, to which all other laws must conform. He maintained
that all Filipinos, whether indigenous or not, are subject to the Constitution. The
concept of ownership under the IPRA still perpetually withdraws such property
from the control of the State and from its enjoyment by other citizens of the
Republic. The perpetual and exclusive character of private respondents claims
simply makes them repugnant to basic fairness and equality. He also asserted that
Carino v. Insular Government was modified by the Constitution.

Finally, he asserted that the law declares ancestral domains, including the natural
resources found therein, are owned by ICCs/IPs and cannot be sold, disposed or
destroyed. This means that while ICCs/IPs could own vast ancestral domains, the
majority of Filipinos who are not indigenous can never own any part thereof.
16

FRANCISCO I. CHAVEZ vs PUBLIC ESTATES AUTHORITY and AMARI


COASTAL BAY DEVELOPMENT CORPORATION
G.R. No. 133250
July 9, 2002

Doctrine:
Under the Regalian doctrine, submerged lands are property of the State and are inalienable.
Pursuant to Article XII of the 1987 Constitution... Submerged lands, like the waters (sea or
bay) above them, are part of the State’s inalienable natural resources. Submerged lands are
property of public dominion, absolutely inalienable and outside the commerce of man. This
is also true with respect to foreshore lands. Any sale of submerged ore foreshore lands is
void being contrary to the Constitution. Reclaimed lands are no longer foreshore or
submerged lands and thus may qualify as alienable agricultural lands.

Relevant Facts:
In 1973, the Commissioner on Public Highways entered into a contract to reclaim
areas of Manila Bay with the Construction and Development Corporation of the
Philippines. Public Estates Authority (PEA) was created by President Marcos under PD
1084, tasked with developing and leasing reclaimed lands. These lands were transferred to
the care of PEA under PD 1085 as part of the Manila Cavite Road and Reclamation Project
(MCRRP). CDCP and PEA entered into an agreement that all future projects under the
MCRRP would be funded and owned by PEA. By 1988, President Aquino issued Special
Patent No. 3517 transferring lands to PEA. It followed by the transfer of 3 Titles (7309,
7311 and 7312) by the Register of Deeds of Paranaque to PEA covering the three
reclaimed islands known as the Freedom Islands.
Subsequently, PEA entered into a joint venture agreement (JVA) with AMARI, a
Thai-Philippine corporation to develop the Freedom Islands. Along with another 250
hectares, PEA and AMARI entered the JVA which would later transfer said lands to
AMARI. This caused a stir especially when Senator Maceda assailed the agreement,
claiming that such lands were part of public domain (famously known as the “mother of all
scams”).
Petitioner Frank Chavez filed a case as a taxpayer praying for mandamus, a writ of
preliminary injunction and a TRO against the sale of reclaimed lands by PEA to AMARI
and from implementing the JVA. Following these events, under President Estrada’s
administration, PEA and AMARI entered into an Amended JVA and MR. Chavez claim
that the contract is null and void.
17

Issue:
Does the stipulations in the Amended JVA for the transfer to AMARI of lands, reclaimed
or to be reclaimed, violate the Constitution.

Holding:
Yes, the Amended JVA that seeks to transfer to AMARI, a private corporation, ownership
of 77.34 hectares of the Freedom Islands is void for being contrary to Section 3, Article XII
of the 1987 Constitution which prohibits the alienation of natural resources other then
agricultural lands of the public domain. Since the Amended JVA also seeks to transfer to
AMARI ownership of 290.156 hectare of still submerged areas of Manila Bay, such
transfer is void for being contrary to Section 2, Article XII of the 1987 Constitution which
prohibits the alienation of natural resources other than agricultural lands of the public
domain. PEA may reclaim these submerged areas. Thereafter, the government can
reclassify the reclaimed lands as alienable or disposable, and further declare them no longer
needed for public service. Still, the transfer of such reclaimed alienable lands of the public
domain to AMARI will be void in view of Section 3, Article XII of the 1987 Constitution
which prohibits private corporations from acquiring any kind of alienable land of the public
domain.
18

FRANCISCO I. CHAVEZ vs PUBLIC ESTATES AUTHORITY and AMARI


COASTAL BAY DEVELOPMENT CORPORATION
G.R. No. 133250
May 6, 2003

Doctrine:
Under the Regalian doctrine, submerged lands are property of the State and are inalienable.
Pursuant to Article XII of the 1987 Constitution... Submerged lands, like the waters (sea or
bay) above them, are part of the State’s inalienable natural resources. Submerged lands are
property of public dominion, absolutely inalienable and outside the commerce of man. This
is also true with respect to foreshore lands. Any sale of submerged ore foreshore lands is
void being contrary to the Constitution. Reclaimed lands are no longer foreshore or
submerged lands and thus may qualify as alienable agricultural lands of the public domain
provided the requirements of public land laws are met.

Relevant Facts:
Several motions for reconsideration of the Supreme Court’s July 9, 2002 decision
which declares the Amended JVA null and void ab initio were filed. The conclusions of
said decisions were summarized by the Court as follows:
The 157.84 hectares of reclaimed lands comprising the Freedom Islands, now covered
by certificates of title in the name of PEA, are alienable lands of the public domain. PEA
may lease these lands to private corporations but may not sell or transfer ownership of
these lands to private corporations. PEA may only sell these lands to Philippine citizens,
subject to the ownership limitations in the 1987 Constitution and existing laws.
The 592.15 hectares of submerged areas of Manila Bay remain inalienable natural
resources of the public domain until classified as alienable or disposable lands open to
disposition and declared no longer needed for public service. The government can make
such classification and declaration only after PEA has reclaimed these submerged areas.
Only then can these lands qualify as agricultural lands of the public domain, which are the
only natural resources the government can alienate. In their present state, the 592.15
hectares of submerged areas are inalienable and outside the commerce of man.
Since the Amended JVA seeks to transfer to AMARI ownership of 77.34 hectares of
the Freedom Islands, such transfer is void for being contrary to Section 3, Article XII of the
1987 Constitution which prohibits private corporations from acquiring any kind of
alienable land of the public domain.
Since the Amended JVA also seeks to transfer to AMARI ownership of 290.156
hectares of still submerged areas of Manila Bay, such transfer is void for being contrary to
Section 2 of Article XII of the 1987 Constitution which prohibits the alienation of natural
resources other than agricultural lands of the public domain. PEA may reclaim these
19

submerged areas. Thereafter, the government can classify the reclaimed lands as alienable
or disposable, and further declare them no longer needed for public service. Still, the
transfer of such reclaimed lands of the public domain to AMARI will be void.
In its Supplement to Motion for Reconsideration, AMARI argues that the Decision
should be made to apply prospectively, not retroactively to cover the Amended JVA.
AMARI also argues that the existence of a statute or executive order prior to its being
adjudged void is an operative fact to which legal consequences are attached citing De
Agbayani v. PNB. AMARI likewise asserts that a new doctrine of the Court cannot operate
retroactively if it impairs vested rights. AMARI maintains that the new doctrine embodied
in the Decision cannot apply retroactively on those who relied on the old doctrine in good
faith, citing Spouses Benzonan v. Court of Appeals.

Issue:
Should the July 9, 2002 ruling of the Supreme Court be reversed?

Holding:
No. Amari’s reliance on De Agbayani and Spouses Benzonan is misplaced. These cases
would apply if the prevailing law or doctrine at the time of the signing of the Amended
JVA was that a private corporation could acquire alienable lands of the public domain, and
the Decision annulled the law or reversed this doctrine. Obviously, this is not the case here.
Under the 1935 Constitution, private corporations were allowed to acquire alienable
lands of the public domain. But since the effectivity of the 1973 Constitution, private
corporations were banned from holding, except by lease, alienable lands of the public
domain. The 1987 Constitution continued this constitutional prohibition. The prevailing
law before, during and after the signing of the Amended JVA is that private corporations
cannot hold, except by lease, alienable lands of the public domain. The Decision has not
annulled or in any way changed the law on this matter. The Decision, whether made
retroactive or not, does not change the law since the Decision merely reiterates the law that
prevailed since the effectivity of the 1973 Constitution. Thus, De Agbayani, which refers to
a law that is invalidated by a decision of the Court, has no application to the instant case.
Amari cannot claim good faith because even before Amari signed the Amended JVA
on March 30, 1999, petitioner had already filed the instant case on April 27, 1998
questioning precisely the qualification of Amari to acquire the Freedom Islands. Even
before the filing of this petition, two Senate Committees14 had already approved on
September 16, 1997 Senate Committee Report No. 560. This Report concluded, after a
well-publicized investigation into PEA’s sale of the Freedom Islands to Amari, that the
Freedom Islands are inalienable lands of the public domain. Thus, Amari signed the
20

Amended JVA knowing and assuming all the attendant risks, including the annulment of
the Amended JVA.
Finally, the Office of the Solicitor General and PEA argue that the cost of reclaiming
deeply submerged areas is "enormous" and "it would be difficult for PEA to accomplish
such project without the participation of private corporations." The Decision does not bar
private corporations from participating in reclamation projects and being paid for their
services in reclaiming lands. What the Decision prohibits, following the explicit
constitutional mandate, is for private corporations to acquire reclaimed lands of the public
domain. There is no prohibition on the directors, officers and stockholders of private
corporations, if they are Filipino citizens, from acquiring at public auction reclaimed
alienable lands of the public domain. They can acquire not more than 12 hectares per
individual, and the land thus acquired becomes private land.
21

SENR, et. al. v. Yap, et. al.,


G.R. No. 167707
October 8, 2008

DOCTRINES:

A positive act declaring land as alienable and disposable is required. In keeping with the
presumption of State ownership, the Court has time and again emphasized that there must be
a positive act of the government, such as an official proclamation, declassifying inalienable
public land into disposable land for agricultural or other purposes. In fact, Section 8 of CA
No. 141 limits alienable or disposable lands only to those lands which have been officially
delimited and classified.

Forest Lands. A forested area classified as forest land of the public domain does not lose
such classification simply because loggers or settlers may have stripped it of its forest
cover. Parcels of land classified as forest land may actually be covered with grass or planted
to crops by kaingin cultivators or other farmers. Forest lands do not have to be on mountains
or in out of the way places. Swampy areas covered by mangrove trees, nipa palms, and other
trees growing in brackish or sea water may also be classified as forest land. The
classification is descriptive of its legal nature or status and does not have to be
descriptive of what the land actually looks like. Unless and until the land classified as
forest is released in an official proclamation to that effect so that it may form part of the
disposable agricultural lands of the public domain, the rules on confirmation of imperfect
title do not apply.

FACTS:

Petitioner-claimants filed a declaratory relief with the RTC of Kalibo, Aklan believing that
Proclamation No. 1801 and PTA Circular 3-82 which declared Boracay Island and others as
tourist zones and marine reserves, precluded them from filing a judicial confirmation of
imperfect title. They contend that since the Island was classified as a tourist zone, it was
susceptible of private ownership and under the Public Land Act and that they had the right to
have the lots registered in their names through judicial confirmation of imperfect titles.

Petitioner Republic of the Philippines, through the Office of the Solicitor General (OSG),
opposed the petition for declaratory relief. The OSG countered that Boracay Island was
an unclassified land of the public domain. Since Boracay Island had not been classified as
alienable and disposable, whatever possession they had cannot ripen into ownership. The
RTC ruled in favour of respondent-claimants which decision was later affirmed by the CA.
Hence, the present petition under Rule 45.
22

In G.R. No. 173775, during the pendency of G.R. No. 167707, President Gloria Macapagal-
Arroyo issued Proclamation No. 1064 classifying Boracay Island into forest land, for
protection purposes, and agricultural land which are alienable and disposable. Petitioners-
claimants and other landowners in Boracay filed with this Court an original petition for
prohibition, mandamus, and nullification of Proclamation No. 1064.

Petitioners-claimants averred that being classified as neither mineral nor timber land, the
island of Boracay is deemed agricultural pursuant to the Philippine Bill of 1902 and Act No.
926, known as the first Public Land Act. Thus, their possession in the concept of owner for
the required period entitled them to judicial confirmation of imperfect title.

OSG opposed andargued that petitioners-claimants do not have a vested right over their
occupied portions in the island. Boracay is an unclassified public forest land pursuant to
Section 3(a) of PD No. 705. Being public forest, the claimed portions of the island are
inalienable and cannot be the subject of judicial confirmation of imperfect title. It is only the
executive department, not the courts, which has authority to reclassify lands of the public
domain into alienable and disposable lands. There is a need for a positive government act in
order to release the lots for disposition.

On November 21, 2006, this Court ordered the consolidation of the two petitions as they
principally involve the same issues on the land classification of Boracay Island.

ISSUES:

1. Did Proclamation No. 1801 and PTA Circular No. 3-82 pose any legal obstacle for
Yap et al and Sacay et al, and all those similarly situated, to acquire title to their
occupied lands in Boracay Island?
2. Is Boracay Island a forest Land? Is it subject to private ownership?

HOLDING:

FIRST ISSUE. Yes. The SC ruled against Yap et al and Sacay et al. The Regalian Doctrine
dictates that all lands of the public domain belong to the State, that the State is the source of
any asserted right to ownership of land and charged with the conservation of such patrimony.
All lands that have not been acquired from the government, either by purchase or by grant,
belong to the State as part of the inalienable public domain.
23

A positive act declaring land as alienable and disposable is required. In the case at bar, no
such proclamation, executive order, administrative action, report, statute, or certification was
presented. The records are bereft of evidence showing that, prior to 2006, the portions of
Boracay occupied by private claimants were subject of a government proclamation that the
land is alienable and disposable. Absent such well-nigh incontrovertible evidence, the Court
cannot accept the submission that lands occupied by private claimants were already open to
disposition before 2006. Matters of land classification or reclassification cannot be assumed.

Private claimants are not entitled to apply for judicial confirmation of imperfect title under
CA No. 141. Neither do they have vested rights over the occupied lands under the said law.
There are two requisites for judicial confirmation of imperfect or incomplete title under CA
No. 141, namely:

(1) open, continuous, exclusive, and notorious possession and occupation of the subject land
by himself or through his predecessors-in-interest under a bona fide claim of ownership since
time immemorial or from June 12, 1945; and

(2) the classification of the land as alienable and disposable land of the public domain.

The tax declarations in the name of private claimants are insufficient to prove the first
element of possession. The SC noted that the earliest of the tax declarations in the name of
private claimants were issued in 1993. Being of recent dates, the tax declarations are not
sufficient to convince this Court that the period of possession and occupation commenced on
June 12, 1945.

The continued possession and considerable investment of private claimants do not


automatically give them a vested right in Boracay. Nor do these give them a right to apply for
a title to the land they are presently occupying. The SC is constitutionally bound to decide
cases based on the evidence presented and the laws applicable. As the law and jurisprudence
stand, private claimants are ineligible to apply for a judicial confirmation of title over their
occupied portions in Boracay even with their continued possession and considerable
investment in the island.

SECOND ISSUE. Yes. Except for lands already covered by existing titles, Boracay was an
unclassified land of the public domain prior to Proclamation No. 1064. Such unclassified
lands are considered public forest under PD No. 705. The DENR and the National Mapping
and Resource Information Authority certify that Boracay Island is an unclassified land of the
public domain.

PD No. 705 issued by President Marcos categorized all unclassified lands of the public
domain as public forest. Section 3(a) of PD No. 705 defines a public forest as a mass of lands
of the public domain which has not been the subject of the present system of classification for
24

the determination of which lands are needed for forest purpose and which are not. Applying
PD No. 705, all unclassified lands, including those in Boracay Island, are ipso
facto considered public forests. PD No. 705, however, respects titles already existing prior to
its effectivity.

However, Boracay, no doubt, has been partly stripped of its forest cover to pave the way for
commercial developments. As a premier tourist destination for local and foreign tourists,
Boracay appears more of a commercial island resort, rather than a forest land. The fact that
the island has already been stripped of its forest cover; or that the implementation of
Proclamation No. 1064 will destroy the islands tourism industry, do not negate its character
as public forest.

The discussion in Heirs of Amunategui v. Director of Forestry is particularly instructive:

A forested area classified as forest land of the public domain does not lose such classification
simply because loggers or settlers may have stripped it of its forest cover. Parcels of land
classified as forest land may actually be covered with grass or planted to crops
by kaingin cultivators or other farmers. Forest lands do not have to be on mountains or in out
of the way places. Swampy areas covered by mangrove trees, nipa palms, and other trees
growing in brackish or sea water may also be classified as forest land. The classification is
descriptive of its legal nature or status and does not have to be descriptive of what the land
actually looks like. Unless and until the land classified as forest is released in an official
proclamation to that effect so that it may form part of the disposable agricultural lands of the
public domain, the rules on confirmation of imperfect title do not apply.

Hence in short, Boracay being a forest land, cannot be the subject of private ownership. A
positive act must be made by the government reclassifying Boracay Island from forest land
as part of the public domain to a land which is alienable and disposable.

WHEREFORE, judgment is rendered as follows:

1. The petition for certiorari in G.R. No. 167707 is GRANTED and the Court of Appeals
Decision in CA-G.R. CV No. 71118 REVERSED AND SET ASIDE.

2. The petition for certiorari in G.R. No. 173775 is DISMISSED for lack of merit.
25

Nancy T. Lorzano v. Juan Tabayag, Jr.


G.R. No. 189647

February 6, 2012

Doctrine:

Once a patent is registered and the corresponding certificate of title is issued, the land
covered by them ceases to be part of the public domain and becomes private property.
Further, the Torrens Title issued pursuant to the patent becomes indefeasible a year after the
issuance of the latter. However, this indefeasibility of a title does not attach to titles secured
by fraud and misrepresentation. Well-settled is the doctrine that the registration of a patent
under the Torrens System does not by itself vest title; it merely confirms the registrants
already existing one. Verily, registration under the Torrens System is not a mode of acquiring
ownership.

Relevant Facts:

The petitioner (Lorzano) and the respondent (Tabayag, Jr.) are two of the children of
the late Juan Tabayag. Tabayag owned a parcel of land situated in Sto. Domingo, Iriga City
(subject property). Right after the burial of their father, the petitioner allegedly requested
from her siblings that she be allowed to take possession of and receive the income generated
by the subject property until after her eldest son could graduate from college to which her
siblings agreed to. After the petitioner’s eldest son finished college, her siblings asked her to
return to them the possession of the subject property so that they could partition it among
themselves. However, the petitioner refused to relinquish her possession of the subject
property claiming that she purchased the subject property from their father as evidenced by a
Deed of Absolute Sale of Real Property to which the respondent contended as the signature
was allegedly forged. The respondent asserted that the said deed of sale was acknowledged
before a person who was not a duly commissioned Notary Public and that the subject
property was already covered by Original Certificate of Title (OCT) No. 1786 pursuant to a
Free Patent issued by the Register of Deeds of Iriga City registered under the name of the
petitioner.

The RTC asserted the nullity of the said deed of sale and yielded to a conclusion that
the signature on the other documents was indeed a forgery. The CA affirmed in toto the
decision of the lower court. Hence, this instant petition for review on certiorari.

Issue:

Considering that the subject property was registered under the petitioner’s name pursuant to a
free patent, is an action for reconveyance in favor of the respondent proper?
26

Holding:

Yes. The general rule is that the subject property, being acquired through a grant of free
patent from the government originally belongs to the public domain. A recognized exception
is that situation where plaintiff-claimant seeks direct reconveyance from defendant public
land unlawfully and in breach of trust titled by him, on the principle of enforcement of a
constructive trust. An action for reconveyance is a legal and equitable remedy granted to the
rightful landowner, whose land was wrongfully or erroneously registered in the name of
another, to compel the registered owner to transfer or reconvey the land to him. A private
individual may bring an action for reconveyance of a parcel of land even if the title thereof
was issued through a free patent since such action does not aim or purport to re-open the
registration proceeding and set aside the decree of registration, but only to show that the
person who secured the registration of the questioned property is not the real owner thereof.
If a patent had already been issued through fraud or mistake and has been registered, the
remedy of a party who has been injured by the fraudulent registration is an action for
reconveyance.

Here, the respondent, in filing the amended complaint for annulment of documents,
reconveyance and damages, was not seeking a reconsideration of the granting of the patent or
the decree issued in the registration proceedings. What the respondent sought was the
reconveyance of the subject property to the heirs of the late Tabayag on account of the fraud
committed by the petitioner. It cannot be gainsaid that the heirs of Tabayag, by themselves
and through their predecessors-in-interest, had already acquired a vested right over the
subject property. An open, continuous, adverse and public possession of a land of the public
domain from time immemorial by a private individual personally and through his
predecessors confers an effective title on said possessors whereby the land ceases to be
public, to become private property, at least by presumption. Hence, the right of the heirs of
Tabayag to ask for the reconveyance of the subject property is irrefutable.
27

Heirs of Reterta v. Spouses Mores and Lopez


G.R. No. 159941
17 August 2011

Reconveyance: Concept and Jurisdiction of the Courts

Facts:
The petitioner Heirs of Reterta initiated an action for Quieting of Title and
Reconveyance and Declaration of Nullity of Affidavit and Sales Certificate with damages,
regarding a parcel of land in Tanza, Cavite. They alleged that the said friar land was granted
to their late father, Teofilo, by virtue of his occupation in the government. They also alleged
that they were in open, continuous, exclusive, and notorious possession and occupation of the
land in the concept of owners, and as successors-in-interest, since the time the land was
granted to Teofilo.

The Heirs further claimed that there was an affidavit executed on 1999 stating, among
others, that Teofilo has waived his rights over the said property in favor of the herein
respondent Spouses which, they allege, led to the fraudulent issuance of a transfer certificate
of title in favor of the respondents, thus, culminating to this action.

In their defense, the respondent Spouses Mores and Lopez interposed the argument
that the Heirs had no legal personality to initiate this action; claiming that this action must be
initiated by the Government through the Office of the Solicitor General; and that the property
in question is a friar land, thus, courts have no jurisdiction over the subject matter and such
must be left with the Bureau of Public Lands

Issue:
Is the action for reconveyance proper?
Are the heirs vested with legal capacity to initiate this action for reconveyance?
Is the regional trial court vested with jurisdiction over reconveyance cases?

Holding:

The Supreme Court held that this action made a good case of Reconveyance. Actions
for reconveyance put into issue allegations of erroneous issuances of certificates of titles
riddled with fraud or are alleged to be invalid, as the Heirs so claimed. The Court cited
Arayata v. Joya explaining that a transfer of title over friar lands may be legally done with
the prior approval and registration by the Chief of the Bureau of Lands. This case calls for
28

the determination of the existence of such approval which is a ground to contest the TCT
issued in favor of the Spouses. Thus, this is a case of reconveyance.

Being an action for reconveyance, the Heirs had legal capacity to initiate this case;
they have shown satisfactorily, based on their claims, that they have acquired title and
superior title over the disputed property, which has acquired its private nature by virtue of a
grant. Should it have been public property at the onset, the respondents would have been
correct, which thus ushers a reversion case instead.

Finally, the RTC has jurisdiction over the case; the respondents claim that the Bureau
of Lands have exclusive jurisdiction over the matter is misplaced. Sections 12 and 18 of Act
No. 1120 provide that the Bureau only has jurisdiction over the administration and
disposition of friar lands; the Bureau ceases to have jurisdiction once the friar land is
disposed in favor of a private person and title is duly issued in favor of that private person.

Thus, in conclusion, this case was remanded back to the regional trial court for further
proceedings.
29

REPUBLIC OF THE PHILIPPINES vs. HEIRS OF AGUSTIN L. ANGELES


G.R. No. 141296
October 7, 2002

DOCTRINE:
Elementary is the rule that prescription and laches will not bar actions filed by the State to
recover its own property acquired through fraud by private individuals.

The Facts:

the late Agustin L. Angeles filed his Free Patent Application situated in Capunitan, Orion,
Bataan, and with an area of 3,578 square meters. By virtue of the said free patent
application, Free Patent No. 265340 was issued in favor of the late Agustin L. Angeles. On
the basis of said free patent, Original Certificate of Title No. 194 was issued and registered in
the name of the late Agustin L. Angeles.

the late Agustin L. Angeles, prior to his death, was able to transfer and convey in favor of his
sister, Emilia L. Angeles (now deceased) the one-half (1/2) northern portion of Lot by means
of a Deed of Absolute Sale that was postdated January 5, 1970, when Agustin L. Angeles
was already dead.

the late Emilia L. Angeles was able to transfer and convey, by way of a Deed of Absolute
Sale he same one-half (1/2) northern portion in favor of her daughter, Luz Gancayco Alvarez.
TCT No. T-43712 was thereafter issued by the Register of Deeds who registered the title of
the lot, on the basis of a half-half share, in the names of the late Agustin L. Angeles and Luz
Gancayco Alvarez.

the Samahang Nayon members and Barangay members of Capunitan, Orion, Bataan,
represented by Elvira E. Manabat filed a Protest before the then Bureau of Lands.

In a formal investigation conducted by the DENR, it was found out that late Agustin L.
Angeles or his predecessors-in-interest, have never occupied nor cultivated the subject land.
prior to and after the issuance of the Free Patent in his name, the same having been in the
actual and continuous occupation by the members of the Samahang Nayon since the prewar
days.

The Issue:

Does Prescription Run Against the State?


30

The Held:

True, a title issued on the basis of a free patent is as indefeasible as one judicially
secured. However, this indefeasibility cannot be a bar to an investigation by the State as to
how such title has been acquired, if the purpose of the investigation is to determine whether
or not fraud has been committed in securing the title. One who succeeds in fraudulently
acquiring title to public land should not be allowed to benefit from it

Elementary is the rule that prescription does not run against the State and its subdivisions.
When the government is the real party in interest, and it is proceeding mainly to assert its
own right to recover its own property, there can as a rule be no defense grounded on laches
or prescription.] Public land fraudulently included in patents or certificates of title may be
recovered or reverted to the State in accordance with Section 101 of the Public Land Act.
The right of reversion or reconveyance to the State is not barred by prescription
31

CARO v. SUCALDITO
G.R. No. 157536
May 16, 2005

Proper party to file in a petition for reconveyance or reversion

RELEVANT FACTS:

Gregorio Caro bought a parcel of land known as Assessor's Lot No. 160 from Ruperto
Gepilano as evidenced by a Deed of Sale in 1953. The said lot was situated in Iloilo City of
about 17.9849 hectares. Thereafter, Gregorio Caro sold a portion of the said lot to his son
Melchor Caro, consisting of 70,124 square meters, and now identified as Lot No. 4512 of the
Cadastral survey of Nueva Valencia. The said father and son executed a Deed of Definite
Sale in 1973 covering Lot No. 4512.

On August 1, 1974, Melchor Caro applied for a free patent before the Bureau of Lands
covering the said area of the property which he bought from his father. The application was,
however, opposed by Deogracias de la Cruz. In 1980, the Regional Director rendered a
Decision canceling the said application.

Caro filed a notice of appeal before the Regional Land Office in Iloilo City, docketed as
MNR Case No. 5207. However, the appeal was dismissed in an Order dated June 29, 1982,
on the ground of failure to file an appeal memorandum within the reglementary period
therefor.

On August 29, 1982, Susana R. Sucaldito, as the buyer of Lot No. 4512, filed an Application
for a Free Patent covering the said lot, and was issued Free Patent No. 597599 Thereafter, on
February 20, 1984, Caro filed a Complaint against Sucaldito for "Annulment of Title,
Decision, Free Patent and/or Recovery of Ownership and/or Possession with Damages". He
later filed an amended complaint, alleging that he was the owner of the subject lot, and had
been in possession of the same "since 1953 and/or even prior thereto in the concept of owner,
adversely, openly, continuously and notoriously."

Caro further alleged that since the issuance of the free patent over the subject lot in favor of
Sucaldito was wrongful and fraudulent, she had no right whatsoever over the subject lot.
Hence, as a "trustee of a constructive trust," she was obliged to return the same to him as the
lawful owner

ISSUE:

Does Caro possess the legal personality to file for the reconveyance of the subject land?
32

HELD:

No, Caro does not legal personality to file for the reconveyance of the subject land.

The Supreme Court ruled that under Section 2, Rule 3 of the Rules of Court, every action
must be prosecuted or defended in the name of the real party-in-interest, or one "who stands
to be benefited or injured by the judgment in the suit." Corollarily, legal standing has been
defined as a personal and substantial interest in the case, such that the party has sustained or
will sustain direct injury as a result of the challenged act. Interest means a material interest in
issue that is affected by the questioned act or instrument, as distinguished from a mere
incidental interest in the question involved.

Clearly then, a suit filed by one who is not a party-in-interest must be dismissed. In this case,
the petitioner, not being the owner of the disputed property but a mere applicant for a free
patent, cannot thus be considered as a party-in-interest with personality to file an action for
reconveyance.

To reiterate, the petitioner is not the proper party to file an action for reconveyance that
would result in the reversion of the land to the government. The petitioner has no personality
to "recover" the property as he has not shown that he is the rightful owner thereof. To
reiterate, the proper party in this case is the Solicitor General, as the subject land was
originally owned by the state.
33

DE DELGADO VS. COURT OF APPEALS


G.R. No. 125728
August 28, 2001

DOCTRINE: An action for reconveyance based on violation of a condition in the Deed of


donation should instituted within ten (10) years from the time of such violation.

Action for reconveyance of registered land based on an implied trust prescribes in ten years
and it is from the date of issuance of such title that the effective assertion of adverse title for
purposes of the statute of limitations is counted.

FACTS:

Carlos Delgado was the absolute owner of a parcel of land with an area of 692,549
square meters. On October 5, 1936, said Carlos Delgado granted and conveyed, by way of
donation or gift with quitclaim, all his rights, title, interest, claim and demand over a portion
of said land consisting of 165,000 square meters in favor of the Commonwealth of the
Philippines or its successors.

The donation provides a condition that when the Commonwealth of the Philippines
no longer needs this parcel of land for any military purposes, then said land shall
automatically revert to the donor or its heirs or assigns.

The donee promptly occupied the donated land and constructed buildings thereon for
military purposes.The donee then registered the property in the name of the Commonwealth
of the Philippines. Upon approval of the application for registration it was found out that an
excess of 33,607 square meters from the total area of the parcels actually donated. Such
apparent excess came allegedly from the neighboring parcels of land also owned by Carlos
Delgado.

Upon declaration of independence on July 4, 1946, the Commonwealth of the


Philippines passed out of existence. It was replaced by the existing Republic of the
Philippines, which took over the subject land and turned portions of it over to the then Civil
Aeronautics Administration (CAA), later renamed Bureau of Air Transportation Office
(ATO).

A petition for reconveyance was filed on December 25, 1970, alleging as ground
therefor the violation of the express condition imposed by the donor. However, the case for
reconveyance was eventually dismissed by the lower court without prejudice on September
26, 1983.
34

On September 28, 1989, the widow and surviving heirs of Jose Delgado filed a new
action for reconveyance with the RTC of Catarman, Northern Samar,praying that the
reconveyance of the donated parcel of land be granted for not complying with the condition
imposed in the deed of donation.

In answer to the complaint, respondent Republic of the Philippines contends that


granting there was a violation of the condition, the action for reconveyance is already barred
by laches, waiver and/or prescription.

ISSUE: Does the action for reconveyance already barred by prescription?

RULING: YES.

Applying Article 1144 (1) of the Civil Code on prescription of actions based on a
written contract, the petitioners herein should have instituted the action for reconveyance
within 10 years from the time the condition in the Deed of Donation was violated. The
earliest date the petitioners knew of the said violation of said condition was on July 4, 1946,
when the Republic, as successor of the Commonwealth of the Philippines, took over the
properties and diverted the property to uses other than that imposed by the donor. As found
by the Court of Appeals, the cause of action of the petitioners has clearly prescribed, having
instituted the action for reconveyance only on December 29, 1970, or 24 years after the
condition was violated. Said action was dismissed by the trial court on September 26, 1983
for failure of petitioners to prosecute the case. The institution of a new action for
reconveyance made on September 28, 1989, does not alter respondent court's conclusion but
in fact bolsters it, for by then, a total of 43 long years were allowed by petitioners to lapse
before instituting the case at bar.

Even if the written communication sent by petitioners sometime in January 1969 and
those made on February 10 and March 16, 1989 can be considered as written extrajudicial
demands made by the creditors, they were nevertheless made way beyond the ten-year period
of prescription stated in the law.

With regard to the alleged excess of 33,607 square meters mistakenly included in the
Original Certificate of Title, such action for its reconveyance has likewise prescribed.

It is now well-settled that an action for reconveyance of registered land based on an


implied trust prescribes in ten years and it is from the date of issuance of such title that the
effective assertion of adverse title for purposes of the statute of limitations is counted.
Petitioners should have taken appropriate legal action seasonably, within the ten years
35

prescriptive period. Since petitioners filed their action belatedly, the court held that they have
also lost any right to the aforesaid portion of land consisting of 33,607 square meters.
36

HEIRS OF VALERIANO S. CONCHA v. SPOUSES LUMOCSO


G.R. No. 15812

December 12, 2007

Doctrine:

An action for reconveyance respects the decree of registration as incontrovertible but seeks
the transfer of property, which has been wrongfully or erroneously registered in other
persons' names, to its rightful and legal owners, or to those who claim to have a better
right. There is no special ground for an action for reconveyance. It is enough that the
aggrieved party has a legal claim on the property superior to that of the registered owner and
that the property has not yet passed to the hands of an innocent purchaser for value.

Relevant Facts:

Petitioners, heirs of spouses Dorotea and Valeriano Concha, Sr., claim to be the rightful
owners of several Lots, all situated in Cogon, Dipolog City. Under Section 48(b) of
Commonwealth Act No. 141 (C.A. No. 141), otherwise known as the Public Land Act.
Respondents are the patent holders and registered owners of the subject lots. The records
show that on August 6, 1997, Valeriano Sr. and his children, petitioners all surnamed
Concha, filed a complaint for Reconveyance and/or Annulment of Title with Damages
against "Spouses Gregorio Lomocso and Bienvenida Guya." Herein respondents. They
sought to annul the Free Patent and the corresponding Original Certificate of Title (OCT)
issued in the name of "Gregorio Lumocso" covering one of the lots.

On September 3, 1999, two separate complaints for Reconveyance with Damages were filed
by petitioners, one is against "Cristita Lomocso Vda. de Daan" and the other is against
"Spouses Jacinto Lomocso and Balbina T. Lomocso"

The three complaints commonly alleged: a) that on May 21, 1958, petitioners' parents
(spouses Valeriano Sr. and Dorotea Concha) acquired by homestead a 24-hectare parcel of
land situated in Cogon, Dipolog City; b) that since 1931, spouses Concha "painstakingly
preserved" the forest in the 24-hectare land, including the excess four (4) hectares "untitled
forest land" located at its eastern portion; c) that they possessed this excess 4 hectares of land
(which consisted of Lot No. 6195, one-hectare portion of Lot No. 6196-A and one-hectare
portion of Lot Nos. 6196-B and 7529-A) "continuously, publicly, notoriously, adversely,
peacefully, in good faith and in concept of the owner since 1931;" d) that they continued
possession and occupation of the 4-hectare land after the death of Dorotea Concha on
December 23, 1992 and Valeriano Sr. on May 12, 1999; e) that the Concha spouses "have
37

preserved the forest trees standing in [the subject lots] to the exclusion of the defendants
(respondents) or other persons since 1931" when respondents, "by force, intimidation, [and]
stealth forcibly entered the premises, illegally cut, collected, [and] disposed" several trees,
and that "the land is private land or that even assuming it was part of the public domain,
plaintiffs had already acquired imperfect title thereto" under Sec. 48(b) of C.A. No. 141, as
amended by Republic Act (R.A.) No. 1942; f)that respondents "surreptitiously" filed free
patent applications over the lots despite their full knowledge that petitioners owned the lots;
g) that the geodetic engineers who conducted the original survey over the lots never informed
them of the
survey to give them an opportunity to oppose respondents' applications; j) that respondents'
free patents and the corresponding OCTs were issued "on account of fraud, deceit, bad faith
and misrepresentation"; and h) that the lots in question have not been transferred to an
innocent purchaser.

Issue/s:

1.) Are the following Complaints of the petitioners in the trial court do not state causes of
action for reconveyance.

Holdings:

No. According to the Court, it held that the reliefs sought by the petitioners in the instant
cases typify an action for reconveyance. The following are also the common allegations in
the three complaints that are sufficient to constitute causes of action for reconveyance. The
trial court correctly held that the instant cases involve actions for reconveyance. An action
for reconveyance respects the decree of registration as incontrovertible but seeks the transfer
of property, which has been wrongfully or erroneously registered in other persons' names, to
its rightful and legal owners, or to those who claim to have a better right. There is no special
ground for an action for reconveyance. It is enough that the aggrieved party has a legal claim
on the property superior to that of the registered owner and that the property has not yet
passed to the hands of an innocent purchaser for value.

In a number of cases, the Court have held that actions for reconveyance of or for cancellation
of title to or to quiet title over real property are actions that fall under the classification of
cases that involve "title to, or possession of, real property, or any interest therein."
38

Cawis, et al. v. Cerilles, et al.


G.R. No. 170207

April 19, 2010

Doctrine: Only the State can institute reversion proceedings pursuant to Sec. 101 of Public
Land Act.

Facts:

On 23 September 1957, the DENR, pursuant to Section 79 of the Public Land Act, approved
the sales patent application of Jose V. Andrada (Andrada) for Lot No. 47 situated within
Holy Ghost Hill Subdivision in Baguio City. Sales Patent No. 1319 was issued to Andrada
upon full payment of the purchase price of the lot.

On 4 August 1969, Republic Act No. 6099 took effect. It provided that subject to certain
conditions, parcels of land within the Holy Ghost Hill Subdivision, which included Lot No.
47, would be sold to the actual occupants without the necessity of a public bidding, in
accordance with the provisions of Republic Act No. 730. Claiming to be the actual occupants
referred to in R.A. No. 6099, petitioners protested the sales patent awarded to Andrada. The
Bureau of Lands denied their protest on the ground that R.A. No. 6099, being of later
passage, could no longer affect the earlier award of sales patent to Andrada.

Sometime in 1987, private respondent Ma. Edeliza S. Peralta (Peralta) purchased Lot No. 47
from Andrada. The Deputy Public Land Inspector, in his final report of investigation, found
that neither Andrada nor Peralta had constructed a residential house on the lot, which was
required in the Order of Award and set as a condition precedent for the issuance of the sales
patent. Apparently, it was Vicente Cawis, one of the petitioners, who had built a house on
Lot No. 47.

Sales Patent No. 1319 was nonetheless transferred to Peralta. In the Order for Transfer of
Sales Rights, the Director of Lands confirmed that before the transfer of the sales patent to
Peralta, Andrada had complied with the construction requirement. On 4 December 1987,
Original Certificate of Title was duly issued in Peralta’s name.

On 8 September 1998, petitioners filed a complaint before the trial court alleging fraud,
deceit, and misrepresentation in the issuance of the sales patent and the original certificate of
title over Lot No. 47. They claimed they had interest in the lot as qualified beneficiaries of
R.A. No. 6099 who met the conditions prescribed in R.A. No. 730. They argued that upon
the enactment of R.A. No. 6099, Andrada’s sales patent was deemed cancelled and revoked
in their favor.
39

Issue:

Do the petitioners have the personality to sue for reversion?

Holding:

No. Section 101 of the Public Land Actclearly states:

SEC. 101. All actions for the reversion to the Government of lands of the public domain or
improvements thereon shall be instituted by the Solicitor General or the officer acting in his
stead, in the proper courts, in the name of the Republic of the Philippines.

Even assuming that private respondent indeed acquired title to Lot No. 47 in bad faith, only
the State can institute reversion proceedings, pursuant to Section 101 of the Public Land
Act. Private persons may not bring an action for reversion or any action which would have
the effect of canceling a land patent and the corresponding certificate of title issued on the
basis of the patent, such that the land covered thereby will again form part of the public
domain. Only the OSG or the officer acting in his stead may do so. Since the title originated
from a grant by the government, its cancellation is a matter between the grantor and the
grantee.

Similarly, in Urquiaga v. CA, the Supreme Court held that there is no need to pass upon any
allegation of actual fraud in the acquisition of a title based on a sales patent. Private persons
have no right or interest over land considered public at the time the sales application was
filed. They have no personality to question the validity of the title. The Supreme Court
further stated that granting, for the sake of argument, that fraud was committed in obtaining
the title, it is the State, in a reversion case, which is the proper party to file the necessary
action.

In this case, it is clear that Lot No. 47 was public land when Andrada filed the sales patent
application. Any subsequent action questioning the validity of the award of sales patent on
the ground of fraud, deceit, or misrepresentation should thus be initiated by the State. The
State has not done so and thus, the validity and regularity of the sales patent as well as the
corresponding original certificate of title issued based on the patent should be upheld.
40

Saad, etc. v. Republic of the Philippines


G.R No. 152570
September 27, 2006

Facts:

Socorro Orcullo filed her application for free patent for lot no. 1434 of Cad-315-D, a
parcel of land with an area of 12.8477 hectares located at Barangay Abugon, Sibonga, Cebu.
The Secretary of Agriculture and Natural Resources issued Free Patent number while the
Registry of Deeds for the province of Cebu issued Original Certificate of Title over the said
lot. Subsequently the lot was sold to SAAD Agro-Industries, Inc., by one of the Orcullos
heirs.

The Republic of the Philippines through the Solicitor General, filed a complaint for
annulment of title and reversion of the lot covered by the free patent to the mass of the public
domain on the ground that the issuance of the free patent and title was irregular and
erroneous and allegedly part of the timber land and forest reserve of Sibonga. Urgello filed a
complaint-in-intervention against the heirs of Orcullo about the alleged illegal cutting of
mangrove trees and construction of dikes within the area covered by the Urgellos Fishpond
Lease Agreement.

The trial court dismissed the complaint, finding that respondent failed to show that the
subject lot is part of Timberland and Forest reserve or that had been classified before the
issuance of the Free Patent and the Original Title. The Court of Appeal reverse the trial court
judgment relying on the map which became the basis of the testimonies of the City
Environment and Natural Resources Office Officer. It held that Timber or Forest Land are
subject to private ownership, unless they were classified as Agricultural Lands. The CA
invalidated the sale of lot to petitioner. However, the Urgellos Fishpond Lease Agreement
continue until its expiration because lease does not pass title to the lessee, but the lease
should not renew. The issuance of the Free Patent and Original Certificate of Title is declared
Null and void abnitio. The lot was reverted to as part of the public domain and to be
classified as Timber.

Petitioner claims that the CA erred in relying on the DENR Officers testimony under P.D
705 took effect on May 19,1975 or long after the issuance of the free patent and title. He
claims that it should not be retroactive to cover and prejudice vested rights acquired prior to
the effectivity of the said law. He also question the reliance of CA on the Land Classification
Map presented by respondent, the fact that it is not certified true copy. He further, contends
that the projection survey conducted by the DENR to determine the subject lot falls within
the forest area used a magnetic box was unreliable and inaccurate instrument. Finally, he
claims that respondent failed to overcome the presumption of regularity of the issuance of the
41

free patent and title in favor of Socorro Orcullo and asserts that respondent failed to show
that the subject lot is inside the timberland block.

Issue: Whether or not the state has sufficiently proved that the land is part of the timberland
and forest reserve at the time respondent was granted free patent and issued a title.

Ruling:

No. The state failed to proved that the land was classified as timberland and forest reserve at
the time respondent was granted free patent and issued a title. The court finds that the trial
court decision is more in accordance with the law rather than the decision of the CA.

The testimony of Isabelo R. Montejo is obviously referring to the P.D No. 705 particularly
section 13. Reliance on this provision is highly misplaced, the fact that P.D No. 705 was
promulgated only on May 19,1975, four years after the free patent and title were awarded to
Orcullo. Thus, it finds no application in the instant case even prior forestry law which was
revised by P.D No. 705 does not contain similar provision. Under article 4 of the Civil Code
provides that the laws shall have no retroactive effects unless the contrary is provided. Thus,
even assuming that the subject parcel of lot was unclassified at the time Orcullo applied for
free patent. It can no longer be considered unclassified and part of the public forest because
the free patent and title were issued thereon in 1971, four years after the effectivity of the P.D
No. 705.

The court observe that the document adverted to is a mere photocopy of the purported
original. A mere photocopy is not qualify as competent evidence of the existence of the L.C.
Map. Under the best evidence rule, the original document must be produced. However,
respondent claims that presentation of the original L.C. Map is unnecessary since it is in the
custody of public officer or in the record of the public office. The court emphasized that all
parties, including the government are bound by the Rules of Admissibility and must comply
with it and must be applied uniformly. Government when it comes to court litigate with one
of its citizen, must submit to the rules of procedure and its rights and privileges at every stage
of the proceedings are substantially in every respect the same as those of its citizens, it
cannot be have a superior advantage.

Obviously, private interest have intervened before classification was made pursuant to
P.D. No. 705. Not only has Orcullo by herself and through her predecessors-in-interest
cultivated and possessed the subject lot since 1930, a free patent was also awarded to her and
a title issued in her name as early as 1971. In fact, it appears that the issuance of the free
patent and certificate of title was regular and in order. Orcullo complied with the requisites
for the acquisition of free patent provided under Commonwealth Act No. 141( Public Land
42

Act), as certified by the Director of Lands and approved by the Secretary of Agriculture and
Natural Resources.
43

Heirs of Nagaño v Court of Appeals


G.R. No. 123231

November 17, 1997

Possession in the concept of an owner

Facts:

Private respondents filed a complaint for the declaration of nullity of the Original Certificate
of Title issued to the heirs of Nagaño, herein petitioners. Respondents claim that the issuance
of title was based on fraud, deceit and misrepresentation committed by Macario Valerio.
Private respondents alleged that part of the subject lot was owned by their predecessors-in-
interest since 1920 but recently discovered that it was fraudulently registered by defendant
Valerio in the name of the heirs of Nagaño. Private respondents asked that the properties be
segregated but petitioners refused.

Issue:

Is the claim of ownership by the private respondents over the subject lot correct?

Ruling:

Yes. It is clear from the allegations in the complaint that private respondents claim over the
subject lot is based in the possession of the property in the concept of an owner, having
possessed it openly, peacefully, publicly, continuously and adversely. Even assuming the
land was part of public domain, respondents already acquired an imperfect title under Section
48 of CA 141. The lot is segregated from public domain because the beneficiary is deemed to
have met all the conditions for the government grant, making it beyond the jurisdiction of the
Director of Lands and could not be a subject of free patent. Further, free patent issued over a
private land is null and void.
44

Dolar v. Barangay Lublub etc.


G.R. 152663
November 18, 2005

Doctrine: If a contract of donation provides for automatic rescission or reversion in case of a


breach of a condition and the donee violates it or fails to comply with it, the property donated
automatically reverts back to the donor without need of any judicial declaration. It is only
when the donee denies the rescission or challenges its propriety that the court can intervene
to conclusively settle whether the resolution was proper.

Facts:
Edgar Dolar and Serafin Jaranilla were co-owners of a parcel of land
 September 16, 1981 they (Edgar and Serafin) donated the parcel of land to a barangay
subject to the condition that it shall be used for the purpose of constructing building
and/or development within five (5) years from the execution of the Deed of Donation.
Otherwise, the deed shall have no force and effect and ownership of the land will revert to
the donor. The barangay captain accepted the donation in behalf of Brgy. Lubkub. The
barangay took possession of the property and allowed the construction of buildings by public
and private entities.

 April 12, 1989, or almost eight (8) years from the contract execution Edgar was
issued the Transfer of Certificate Title.
Sometime in June 1989, Edgar executed another deed donating to Brgy. Lublub, the very
same he and Serafin Jaranilla had earlier donated to the same done. It contained exactly the
same conditions expressly set forth in the first.

Edgar filed a complaint for quieting of title and recovery of possession of the area donated
against the barangay claiming that the donation had ceased to be effective, for failure to
comply with the conditions of the donation.

Issue/s:

1. Was the action to quiet title properly made?


2. Can the rule on imprescriptibility of actions to quiet title admits of exceptions?
3. Was the Deed of Donation valid even if it is not registered in the Registry of Deeds?
4. Does Barangay Lublub have the right to acquire the subject property by acquisitive
prescription?
45

Holding/s:

1. No. The action to quiet title is unavailing until the donation shall have first been
revoked. In the case at bar, the barangay traces its claim of ownership over the
disputed property to a valid contact of donation which is yet to be effectively
revoked. Such rightful claim does not constitute a cloud on the supposed title of
Edgardo over the same property removable by an action to quiet title.
2. No. In the case at bar, Edgar’s right of action to revoke or cancel the donation had
indeed prescribed regardless of the applicable provision of the Civil Code.
3. Yes. For, as between the parties to the donation and their assigns, the registration of
the deed with the Registry of Deeds is not needed for its validity and efficacy.
4. Yes. Barangay Lublub anchors its title and right over the donated lot. First and
foremost, by virtue of deed of donation. Admittey, standing alone, adverse,
continuous and long possession of a piece of real property cannot defeat the title of a
registered owner. But this presupposes a Torrens title was lawfully acquired and
issued to the Barangay.
46

East Asia Traders, Inc. v Republic of the Philippines, represented by the Director,
Lands Management Bureau
G.R. 152947

July 7, 2004

DOCTRINE: Public land fraudulently included in patents or certificates of title may be


recovered or reverted to the State in accordance with Section 101 of the Public Land
Act. Prescription does not lie against the State in such cases for the Statute of Limitations
does not run against the State. The right of reversion or reconveyance to the State is not
barred by prescription.

RELEVANT FACTS: On December 15, 1986, Galileo Landicho filed with the Bureau of
Lands, District Office at Lemery, Batangas, Free Patent Application No. 1427. This
application[3] covers Lot No. 4355 consisting of 00.1312 hectare situated in Niogan, Laurel,
Batangas. On March 6, 1987, then Acting District Land Officer Constante Asuncion,
approved[4] the application and issued Free Patent No. 1516 in Landichos
name. Subsequently or on January 22, 1988, the Registry of Deeds of Tanauan, Batangas
issued to him Original Certificate of Title (OCT) No. P-3218.

On June 7, 1989, Landicho sold the lot to Teresita Reyes. Forthwith, Landichos OCT
No. P-3218 was cancelled by the same Registry of Deeds and in lieu thereof, TCT No. 36341
was issued in the name of Teresita Reyes. In turn, on June 7, 1990, Reyes sold the same lot to
East Asia Traders, Inc., petitioner, represented by its Vice-President, Betty Roxas
Chua.Consequently, the Register of Deeds cancelled TCT No. 36341 in the name of Reyes
and in lieu thereof, issued TCT No. 38609 in the name of petitioner.

Meanwhile, the Department of Environment and Natural Resources (DENR),


pursuant to Section 91 of Commonwealth Act No. 141, as amended,[5] conducted an
investigation to ascertain the truth of the material facts alleged in various free patent
applications or whether they are maintained and preserved in good faith. The DENR found
that at the time Landicho applied for a free patent, Lot 4355 was inalienable, being a property
of public dominion intended to be used as a national road.

This prompted the Republic of the Philippines, through the Director of the Lands
Management Bureau to file, on March 9, 1998, with the Regional Trial Court (RTC), Branch
83, Tanauan, Batangas, a complaint for reversion and cancellation of Free Patent No. 1516,
OCT No. P-3218 and its derivative titles (TCT No. 36341 and TCT No. 38609). Impleaded
as defendants were petitioner East Asia Traders, Inc., Landicho, Reyes, and the Register of
Deeds of Tanauan, Batangas.
47

ISSUE: Is the action for reversion filed by respondent Republic of the Philippines already
barred by prescription since it only filed the action for reversion on March 9, 1998, eleven
(11) years after the registration of the land in question?

HOLDING: No, the Court reiterated that prescription does not run against the government.
When the government is the real party in interest, and is proceeding mainly to assert its own
rights and recover its own property, there can be no defense on the ground of laches or
limitation. In addition, Article 1113 of the Civil Code states that all things which are within
the commerce of men, are susceptible of prescription, unless otherwise provided. Property of
the State or any of its subdivisions not patrimonial in character shall not be the object of
prescription.
48

Estate of the Late Yujuico v. Republic of the Philippine


G.R. No. 168661
October 26, 2007

DOCTRINE:

An action for reversion seeks to restore public land fraudulently awarded and disposed of to
private individuals or corporations to the mass of public domain.

An action to recover lands of the public domain is imprescriptible. Such right however can be
barred by laches/estoppel under Sec. 32 of P.D. 1529 which recognizes the rights of innocent
purchasers for value above the interests of the government (Equitable Estopple)

FACTS:

Fermina Marcos filed for an application for the registration and confirmation of her title over
a parcel of land which was granted by the court. The land was then sold to Jesus S. Yujuico
which was subdivided into two lots. Transfer certificates of title was issued in his name and
in the name Augusto Carpio.

Sometime in 1977, Presidential Decree No. (PD) 1085 entitled Conveying the Land
Reclaimed in the Foreshore and Offshore of the Manila Bay (The Manila-Cavite Coastal
Road Project) as Property of the Public Estates Authority as well as Rights and Interests with
Assumptions of Obligations in the Reclamation Contract Covering Areas of the Manila Bay
between the Republic of the Philippines and the Construction and Development Corporation
of the Philippines (1977) was issued. Land reclaimed in the foreshore and offshore areas
of Manila Bay became the properties of the Public Estates Authority (PEA). The PEA also
acquired ownership of other parcels of land along the Manila Baycoast, some of which were
subsequently sold to the Manila Bay Development Corporation (MBDC).

The PEA undertook the construction of the Manila Coastal Road. As this was being planned,
Yujuico and Carpio discovered that a verification survey they commissioned showed that the
road directly overlapped their property, and that they owned a portion of the land sold by the
PEA to the MBDC.

Yujuico and Carpio filed before the Paraaque City Regional Trial Court (RTC), a complaint
for the Removal of Cloud and Annulment of Title with Damages against the PEA wherein
the parties entered into a compromise agreement approved by the trial court

On June 8, 2001, respondent Republic of the Philippines, through the OSG, filed a complaint
alleging that when the land registered to Castro, it was still a portion of Manila Bay therefor
it cannot be registered in the name of Fermina Castro.
49

ISSUE:

Will the action for reversion prosper?

HELD:

No. It was held by the court that upon effectivity of Batas Pambansa (BP) Blg. 129 the
Intermediate Appellate Court has the exclusive original jurisdiction over actions for
annulment of judgments of RTCs.

An action for reversion to cancel titles derived from homestead patents or free patents based
on transfers and conveyances in violation of CA No. 141 is filed by the OSG pursuant to its
authority under the Administrative Code with the RTC. It is clear therefore that reversion
suits were originally utilized to annul titles or patents administratively issued by the Director
of the Land Management Bureau or the Secretary of the DENR.

It is clear therefore that the reversion suit was erroneously instituted in the Paraaque RTC
and should have been dismissed for lack of jurisdiction. The proper court is the CA which is
the body mandated by BP Blg. 129 and prescribed by Rule 47 to handle annulment of
judgments of RTCs.

Assuming that the Paraaque RTC has jurisdiction over the reversion case, still the lapse of
almost three decades in filing the instant case, the inexplicable lack of action of the Republic
and the injury this would cause constrain the court to rule for petitioners. While it may be
true that estoppel does not operate against the state or its agents, deviations have been
allowed.

Equitable estoppel may be invoked against public authorities when as in this case, the lot was
already alienated to innocent buyers for value and the government did not undertake any act
to contest the title for an unreasonable length of time.

When respondent government filed the reversion case in 2001, 27 years had already elapsed
from the time the late Jesus Yujuico purchased the land from the original owner Castro. After
the issuance of OCT No. 10215 to Castro, no further action was taken by the government to
question the issuance of the title to Castro until the case of Public Estates Authority, brought
up in the oral argument before this Court on September 6, 2000. Considering that innocent
purchaser for value Yujuico bought the lot in 1974, and more than 27 years had elapsed
before the action for reversion was filed, then said action is now barred by laches.
50

Sta. Ignacia Rural Bank, Inc. v. Court of Appeals


G.R. 97872
March 1, 1994

DOCTRINE:

The right to repurchase under Section 119 cannot be waived by the party entitled thereto, and
applies with equal force to both voluntary and involuntary conveyances. Such right is
available in foreclosure sales of lands covered by homestead or free patent.

FACTS:

Sta. Ignacia Rural Bank, Inc. extended to Spouses Conrado Pablo and Juanita Gonzales a
loan totalling P12,109.75 which was secured by a residential house and two lots. The spouses
defaulted payment which resulted to extrajudicial foreclosure of their real estate mortgate.
The house and lot was sold at a public auction where the defendant bank was the highest
bidder on July 28, 1981.

Thereafter, the Certificate of Sale was executed in favor of the defendant bank on September
29, 1981 and the same was registered with the Register of Deeds on November 5, 1981. On
December 19, 1984, the defendant bank sold the aforementioned real estates to defendant-
spouses Alberto Lucas and Nelia Rico and Transfer Certificates of Title over the house and
lots were subsequently issued in the name of said defendant-spouses.

A complaint for the repurchase of the subject house and lots, annulment of title and damages
by the plaintiff-spouses.

ISSUE:

Does the right to repurchase the subject house and lots already prescribed?

HELD:

No. The Court held that rules on redemption in the case of an extrajudicial foreclosure of
land acquired under free patent or homestead statutes may be summarized as follows: If the
land is mortgaged to a rural bank under R.A. No. 720, as amended, the mortgagor may
redeem the property within two (2) years from the date of foreclosure or from the registration
of the sheriff's certificate of sale at such foreclosure if the property is not covered or is
covered, respectively, by a Torrens title. If the mortgagor fails to exercise such right, he or
his heirs may still repurchase the property within five (5) years from the expiration of the two
(2) year redemption period pursuant to Section 119 of the Public Land Act (C.A. No. 141). If
the land is mortgaged to parties other than rural banks, the mortgagor may redeem the
property within one (1) year from the registration of the certificate of sale pursuant to Act
No. 3135. If he fails to do so, he or his heirs may repurchase the property within five (5)
51

years from the expiration of the redemption period also pursuant to Section 119 of the Public
Land Act.

In the case, the certificate of sale in favor of petitioner was registered with the Register of
Deeds on November 5, 1981, private respondents had two years, reckoned from said date,
within which to redeem the property from petitioner, and another five years, under
Commonwealth Act No. 141, counted from the expiration of the redemption period, to effect
repurchase which private respondents precisely did when the suit below was initiated on
March 20, 1986.
52

Mejia v. Gabayan, et. al.


G.R. No. 149756,
April 12, 2005

Doctrine

Homestead patents are not exempt from the operation of land reform specially that is
devoted to rice or corn crops

Relevant Facts

Arturo Mejia is the registered owner of a parcel of land located in Sinamar, San Mateo,
Isabela covered by Transfer Certificate of Title (TCT) No. 75164 which was issued on July
3, 1974. The lot was a portion of a large tract of land covered by a homestead patent granted
by the President of the Philippines to Dalmacio Mejia on December 11, 1936, on the basis of
which Original Certificate of Title (OCT) No. T-4184 was issued by the Register of Deeds of
Isabela.[1]

On August 13, 1978, the President of the Philippines, through the Secretary of Agrarian
Reform, issued Certificates of Land Transfer (CLT) over portions of the property covered by
TCT No. 75164 to 5 different beneficiaries: Mejia filed a petition with the Department of
Agrarian Reform (DAR) in 1983 for their cancellation. However, no action was taken on the
said petition.[3]

On May 10, 1993, Mejia filed a petition with the DAR, for the exclusion of the property from
PD No. 27 which was denied. Mejia appealed the Order to the DAR Regional Director. On
June 20, 1994 Mejia opted to take advantage of the ruling of this Court in Alita v. Court of
Appeals wherein it was held that properties covered by homestead patents were not covered
by Presidential Decree (PD) No. 27, he filed a complaint in the Regional Trial Court (RTC)
of Isabela against the respondents Filomena Gabayan, Albin Rueme, Ernesto Mejia, Carlos
Ramos, Josefina Lacadin and Pedro Gavino, for declaratory relief, and for the recovery of the
possession of the property. On May 23, 1995, the DAR Regional Director issued an Order
granting the petition of Mejia exempting his property from the Comprehensive Agrarian
Reform Program (CARP), but ordering him to allow the respondents-tenants to remain in
possession of the property and to execute Leasehold Contracts in their favor in accordance
with Rep. Act No. 3844. The trial court ruled that the Order of the DAR Secretary must be
respected. The Secretary of Agrarian Reform issued an Order dated December 27,
1996affirmed the resolution of the Regional Director with modification:
53

1. Only that portion of TCT No. T-75164 which petitioner is personally cultivating (3.6691
hectares) shall be exempt from P.D. No. 27 and R.A. No. 6657, thus constituting as part of
his retained area;

2. The petitioner may choose from the tenanted portion as an addition to compromise his
aggregate area of retention which should not exceed five (5) hectares; and

3. CLTs/Eps recalled or cancelled by the previous Order shall be re-issued (however this
time as CLOAs since the subject landholding is a homestead land not covered by P.D. No.
27), except those within the landowner is retained area which shall be under the leasehold
system. The trial court ruled that the December 27, 1996 Order of the DAR Secretary must
be respected.

Issue/s

A. Are the subject properties covered by homestead patents exempt from the operation
of land reform?

B. Is petitioner entitled to retain seven hectares?

Holding/s

A. No. Presidential Decree (PD) No. 27, under which the Emancipation Patents
sought to be cancelled here that were issued to respondents, applies to all tenanted private
agricultural lands primarily devoted to rice and corn under a system of share-crop or lease-
tenancy, whether classified as landed estate or not. The law makes no exceptions whatsoever
in its coverage. Nowhere therein does it appear that lots obtained by homestead patents are
exempt from its operation.

The matter is made even clearer by Department Memorandum Circular No. 2, Series
of 1978, which states: Tenanted private agricultural lands primarily devoted to rice and/or
corn which have been acquired under the provisions of Commonwealth Act 141, as amended,
shall also be covered by Operation Land Transfer. Unquestionably, petitioners parcels of
land, through obtained by homestead patents under Commonwealth Act 141, are covered by
land reform under PD 27.

B. No. PD 27, which provides the retention limit, states:

In all cases, the landowner may retain an area of not more than seven (7) hectares if
such landowner is cultivating such area or will now cultivate it. In the case at bar, neither of
54

the conditions for retention is present. Homestead grantees or their direct compulsory heirs
can own and retain the original homesteads, only for as long as they continue to
cultivate them. That parcels of land are covered by homestead patents will not automatically
exempt them from the operation of land reform. It is the fact of continued cultivation by the
original grantees or their direct compulsory heirs that shall exempt their lands from land
reform coverage.

In the present case, neither petitioner nor her heirs are personally cultivating the
subject homesteads. The DAR and the CA found that respondents were the ones who had
been cultivating their respective portions of the disputed properties. Under the circumstances,
the only right of the petitioner is to retain five (5) hectares of the subject property, in
accordance with Section 6 of Rep. Act No. 6657.
55

Kings Properties Corp. v. Galido


G.R. 170023

November 27, 2009

Doctrine: Under CA 141, in purchasing lands of homestead patents, several conditions and
requirements are to be followed such as an approval by the SENR of the sale.

Facts: The Heirs of Domingo Eniceo; Rufina and Maria Eniceo was awarded with
homestead patent No. 112947 consisting of 4 parcels of land in Antipolo, Rizal on April 18,
1966 which was registered under Original Certificate Tile No. 535. The issuance of the said
patent was subject to conditions to which, it was subject to the provisions of sections 118,
121, 122 and 124 of Commonwealth Act No. 141. Here it provided that except in favor of the
Government or any of its branches, units or institutions, the land hereby acquired shall be
inalienable and shall not be subject to incumbrance for a period of 5 years next following the
date of this patent, and shall not be liable for the satisfaction of any debt contracted prior to
the expiration of the period; that it shall not be alienated , transferred, or conveyed after 5
years and before 25 years next following the issuance of title, without the approval of
Secretary of Agriculture and Natural Resources, that it shall not be incumbered, alienated, or
transferred to any person, corporation, association or partnership not qualified to acquire
public lands under the said Act and its amendments. On September 10, 1973, or 7 years after
the award of the patent, a deed of sale covering the property in Antipolo was executed
between the Eniceos and Respondent, Galido. A duplicate copy of the OCT was delivered to
the respondent. By 1975a notice of loss and with the registry of deeds of Marikina City a
notice of loss and subsequently filed a petition for the issuance of a new owners’ duplicate
copy of the OCT. One year after, the RTC ruled that the certified try copy of the OCT
contained no annotation in favor of any person, corporation or entity. IT was ordered that a
second owners copy be released in favor of Eniceo and declared the original owners of the
OCT cancelled and to be of no value. On March 20, 1995, Eniceo heirs executed a deed of
absolute sale in favor of the petitioner covering lots 3 and 4 of the property to which an OCT
was alter issued. Lots 1 and 5 was registered in the names of Eniceo and Bolinas. On April 5,
1995, Eniceo heirs executed another deed of sale covering lots 1 and 5 in favor of the
petitioner, while the first TCT involving lots 3 and 4 are cancelled. On August 17, 1995, the
SENR approved the deed of sale between Eniceo and the Respondent. By January 16, 1996,
respondent filed a complaint for cancellation of the certificates of title issued in favor of the
petitioner, and the registration of the deed of sale and issuance of a new transfer of title in
favor of respondent. The complaint was dismissed by the RTC although the CA reversed the
first decision.

Issues: Should the subsequent sale of the Eniceo heirs in favor of the petitioner be
considered to be valid?
56

Ruling: No, the court affirmed the decision of the Court of Appeals to which it declared the
TCT between the Petitioner and the Heirs to be null and void. Firstly, the CA pointed out the
fact that the sale is a notarized document to which it favors the presumption of regularity. In
addition, the Supreme Court also stressed out the requirement of the approval of the SENR
which was made in favor of the respondent, to which no alienation, transfer, or conveyance
of any homestead after 5 years and before 25 years after the issuance of title shall be valid
without the approval of the SENR. Also, the court declared the petitioner to be estopped from
questioning the deed of sale in favor of the respondent for merely assuming the rights of the
heirs. The court added the Alfredo vs Spouses Borras that discussed the requirement of the
approval of the SENR Secretary. As such, the approval maybe secured later, which is what
happened in favor of the respondent as the approval was made more than 20 years later. This
did not ipso facto make a sale void. Lastly, the court added the principle prius tempore or
first in time stronger in right. Meaning, the respondent, being the first taker, has stronger
rights rather than that of the respondent who was under the questioned subsequent sale.
Thus, the decision of Court of Appeals in favor of Galido is affirmed.
57

Republic of the Philippines vs. Court of Appeals


G.R. No. 100709
November 14, 1997

Doctrine

Sections 118 to 124 of the Public Land Act clearly proscribe the encumbrance of a parcel of
land acquired under a free patent or homestead within five years from the grant of such
patent. Furthermore, such encumbrance results in the cancellation of the grant and the
reversion of the land to the public domain. Encumbrance has been defined as anything that
impairs the use or transfer of property; anything which constitutes a burden on the title; a
burden or charge upon property; a claim or lien upon property. It may be a legal claim on an
estate for the discharge of which the estate is liable; an embarrassment of the estate or
property so that it cannot be disposed of without being subject to it; an estate, interest, or
right in lands, diminishing their value to the general owner; a liability resting upon an estate.

Relevant Facts

Respondent Morato filed a Free Patent Application on a parcel of land which was approved
and the Register of Deeds issued an Original Certificate of Title. Both the free patent and the
title specifically mandate that the land shall not be alienated nor encumbered within five (5)
years from the date of the issuance of the patent.

Subsequently, the District Land Officer in Lucena City, acting upon reports that respondent
Morato had encumbered the land in violation of the condition of the patent, conducted an
investigation. Thereafter, it was established that the subject land is a portion of the Calauag
Bay, five (5) to six (6) feet deep under water during high tide and two (2) feet deep at low
tide, and not suitable to vegetation. Moreover, a portion of the land was mortgaged by
respondent Morato to respondents Nenita Co and Antonio Quilatan. The spouses Quilatan
constructed a house on the land. Another portion of the land was leased to Perfecto
Advincula, where a warehouse was constructed.

Petitioner filed an amended complaint against respondents Morato, spouses Nenita Co and
Antonio Quilatan, and the Register of Deeds for the cancellation of title and reversion of a
parcel of land to the public domain.

RTC dismissed the complaint. CA affirmed.

Issues

1.) Did the respondent violate the free patent condition prohibiting encumbering the land
within the 5-year period?
58

2.) Is the questioned land a part of disposable public land and not a foreshore land?

Holdings

1.) Yes. Public Land Act Sec. 18 provides that “..lands acquired under free patent or
homestead provisions shall not be subject to encumbrance or alienation from the date
of the approval of the application and for a term of five years from and after the date
of issuance of the patent or grant.” Within the five-year prohibitory period,
Respondent Morato mortgaged a portion of the land to Respondent Nenita Co and
leased a portion of the land to Perfecto Advincula. Such contracts of lease and
mortgage executed within five (5) years from the issuance of the patent constitute an
encumbrance and violate the terms and conditions of such patent as contemplated by
section 118 of the Public Land Act.

2.) No. The questioned land is a foreshore land. Article 1, case 3, of the Law of Waters,
provides those which are part of the national domain open to public use: “3. The
Shores. By the shore is understood that space covered and uncovered by the
movement of the tide. Its interior or terrestrial limit is the line reached by the highest
equinoctal tides. Where the tides are not appreciable, the shore begins on the land side
at the line reached by the sea during ordinary storms or tempests.”

In the present case, through the encroachment or erosion by the ebb and flow of the tide, a
portion of the subject land was invaded by the waves and sea advances. The Calauag Bay
shore has extended up to a portion of the questioned land. While at the time of the grant of
free patent to respondent Morato, the land was not reached by the water, however, due to
gradual sinking of the land caused by natural calamities, the sea advances had permanently
invaded a portion of subject land. When the sea moved towards the estate and the tide
invaded it, the invaded property became foreshore land and passed to the realm of the public
domain. In fact, the Court in Government vs. Cabangis annulled the registration of land
subject of cadastral proceedings when the parcel subsequently became foreshore land. The
subject land in this case, being foreshore land, should therefore be returned to the public
domain.
59

LOPEZ V. CA
GR No. 127827

March 5, 2003

DOCTRINE:

Homestead settlement is one of the modes by which public lands suitable for agricultural
purposes are disposed of. Its object is to provide a home for each citizen of the state, where
his family may shelter and live beyond the reach of financial misfortune, and to inculcate in
individuals those feelings of independence which are essential to the maintenance of free
institutions.

A person who is legally qualified has to file his application for a homestead patent with the
Bureau of Lands. If in order, the application shall be approved by the Director, The applicant
will be authorized to enter the land upon payment of an entry fee of five pesos. Within six
months after approval of the application, the applicant has to improve and cultivate the land.
He must cultivate at least one-fifth of the land for a period of not less than two years nor
more than five years from the date of approval of the application. He must also continuously
reside in the same municipality where the homestead is located, or in an adjacent
municipality, for at least one year. He must finally present his final proof to the Bureau of
Lands that he has complied with the cultivation and residency requirements. Act No. 2874
requires that for an application to be valid, it must be approved by the Director of Lands.

FACTS:

In 1920, Fermin Lopez occupied, possessed, and declared for taxation purposes a parcel of
public land containing an area of 19 hectares, 48 ares, 88 centares, more or less, situated in
Makatubong, Barrio De la Paz, Antipolo, Rizal. He filed a homestead application over the
land, but his application was not acted upon until his death in 1934. After Fermin's death,.
Hermogenes then filed a homestead application in his own name when he learned that the
application of their father was not yet acted upon. After ascertaining that the land was free
from claim of any private person, the Bureau approved his application. The land was
surveyed and a resulting plan, H-138612, was approved by the Director of Lands, who
thereafter ordered the issuance of the homestead patent. The patent was later transmitted to
the Register of Deeds of Rizal for transcription and issuance of the corresponding certificate
of title in his name.

Unaware that he has been awarded a homestead patent, Hermogenes executed on February
11, 1956 an Extra-judicial Partition of the disputed land with his brothers; petitioner
Eleuterio, Juan, and Nazario; however, the three executed a Deed of Absolute Sale of their
share in the land in favor of Hermogenes. Hermogenes then applied for the registration of the
60

property in his name. To his surprise, he found that the land has been registered in the names
of Fernando Gorospe, Salvador de Tagle, Rosario de Tagle, Beatriz de Suzuarrequi and
Eduardo Santos, who collectively opposed his application. He filed a complaint for the
annulment of the free patent and title against these persons which was dismissed because
Hermogenes was not a real party in interest since he previously sold his right to the land to
one Ambrocio Aguilar on July 31, 1959.

Respondent Lopezes, as heirs of Hermogenes, filed a complaint against Aguilar before the
RTC for the cancellation of the deed of sale executed by Hermogenes in favor of Aguilar
and/or reconveyance wherein it has been declared the deed of absolute sale null and void ab
initio and the respondents as the true and absolute owner of the disputed land. Aguilar appeal
and motion for reconsideration was then denied.

On May 31, 1985, petitioners Eleuterio, Anatalia, Joselito, Rogelio, Evangeline and Noel, all
heirs of Nazario Lopez, along with Guillermo, Lorenzo, Domingo, Amado, and Victoria, all
heirs of Juan Lopez, instituted the present action against the respondents before the RTC of
Antipolo, Rizal praying that they be declared co-owners of the property subject matter hereof
and that private respondents be ordered to reconvey to them 3/5 thereof as its co-owners, or
in the alternative, to pay its value. The RTC then rendered a decision ordering the dismissal
of the case and declaring Hermogenes Lopez as the exclusive owner of the property in
question. On appeal, the Court of Appeals affirms the RTC ruling but deleting the payment
of attorney’s fees.

ISSUE/S:

Is Fermin Lopez entitled to the grant of the homestead patent?

Are the petitioners the co-owners of the disputed property?

HOLDINGS:

No. The application of Fermin unfortunately remained unacted upon up to the time of his
death. It was neither approved nor denied by the Director, as the Bureau failed to process
it. Hence, he could not have acquired any vested rights as a homestead applicant over the
property because his application was never acted upon.

No. A valid application is sadly lacking in the case of Fermin. This circumstance prevented
him from acquiring any vested right over the land and fully owning it at the time of his
death. Conformably, his heirs did not inherit any property right from him.The failure of the
Bureau of Lands to act on the application of Fermin up to the time of his death, however,
prevented his heirs to be subrogated in all his rights and obligations with respect to the land
applied for. Perforce, at the time Hermogenes applied for a homestead grant over the
61

disputed property, it was still part of alienable public land. As he applied for it in his own
name, his application inures to his sole benefit. After complying with the cultivation and
residency requirements, he became a grantee of a homestead patent over it, thereby making
him its absolute and exclusive owner. They are not its co-owners as it is owned absolutely by
Hermogenes. Well to note, the extra-judicial partition and the special power of attorney to
sell were executed on the mistaken assumption that Hermogenes and his brothers inherited
the property from Fermin. Moreover, at the time the documents were made, Hermogenes was
unaware that he was granted a homestead patent. As correctly ruled by the appellate court,
estoppel does not operate to confer property rights where there are none.

S-ar putea să vă placă și